Complicated Curve Sketch and Derivative Computation (Calc I)

Click For Summary
The discussion centers on challenges faced in calculating limits and derivatives in calculus, particularly with absolute value functions and slant asymptotes. The original poster expresses confusion about determining the domain, range, and intercepts, as well as the need for case analysis when taking derivatives. A response clarifies that slant asymptotes are not necessary for solving the problem, emphasizing the importance of calculating specific limits instead. It also reassures that when taking limits to positive infinity, the absolute value can be simplified. Overall, the focus is on understanding the fundamental concepts of limits and derivatives in calculus.
ShangMing
Messages
4
Reaction score
0

Homework Statement


wvzi4p.png



Homework Equations


Fundamental theorem of calculus.

The Attempt at a Solution


For the first problem, I'm completely lost. I've only worked with slant asymptotes for polynomials so finding the domain, range and intercepts is as far as I've gotten.

I'm not too great at absolute value functions either, so when I take the derivative, do I need to use two cases for positive and negative x when I'm finding intervals of increase/decrease, concavity, etc?

For the second question, I have absolutely no idea where to even begin.

Thanks PF
 
Physics news on Phys.org
You don't need to know anything about slant asymptotes to answer this question. All they're asking you is to calculate two limits:

m=\lim_{x\rightarrow \infty}{\frac{f(x)}{x}}

and once you've calculates that, you need to calculate

\lim_{x\rightarrow \infty}{f(x)-mx}


And don't worry about the absolute value. You take the limit to positive infinity. This means that your x will become very large, and in particular it will become larger then -2. This means that |x+2| becomes positive. So you can drop the absolute value signs.
 
Question: A clock's minute hand has length 4 and its hour hand has length 3. What is the distance between the tips at the moment when it is increasing most rapidly?(Putnam Exam Question) Answer: Making assumption that both the hands moves at constant angular velocities, the answer is ## \sqrt{7} .## But don't you think this assumption is somewhat doubtful and wrong?

Similar threads

Replies
4
Views
2K
  • · Replies 2 ·
Replies
2
Views
2K
  • · Replies 3 ·
Replies
3
Views
2K
  • · Replies 1 ·
Replies
1
Views
1K
  • · Replies 4 ·
Replies
4
Views
3K
  • · Replies 1 ·
Replies
1
Views
2K
  • · Replies 3 ·
Replies
3
Views
2K
  • · Replies 5 ·
Replies
5
Views
2K
  • · Replies 4 ·
Replies
4
Views
2K
Replies
6
Views
2K